K
Khách

Hãy nhập câu hỏi của bạn vào đây, nếu là tài khoản VIP, bạn sẽ được ưu tiên trả lời.

22 tháng 3 2020

A B O M

- Ta có : AB có O là trung điểm .

=> \(OA=OB=\frac{1}{2}AB=\frac{1}{2}6=3\left(cm\right)\)

TH1 : \(M\in OA\)

=> \(AM+MO=3\)

\(MO=1cm\)

=> AM = 2 cm .

\(AM+MB=AB\)

=> \(MB=4\left(cm\right)\)

TH1 : \(M\in OB\)

=> \(BM+MO=3\)

\(MO=1cm\)

=> BM = 2 cm .

\(BM+MA=AB\)

=> \(MA=4\left(cm\right)\)

Bài 6 :

a, Sắp xếp : \(\widehat{A}-\widehat{B}-\widehat{E}-\widehat{D}-\widehat{C}\)

b, Các góc nhọn : \(\widehat{A}-\widehat{B}\)

3 tháng 7 2021

Bài 6: Cho các góc A = 40 độ; B = 60 độ; C = 130 độ; D = 100 độ; E = 90 độ

a) Sắp xếp các góc trên theo thứ tự từ nhỏ đến lớn

\(A< B< E< D< C\)

b) Kể tên các góc nhọn, các góc 

+) Các góc nhọn:  \(A;B\)

+) Các góc tù: \(D;C\)

+) Góc vuông: \(E\)

Giải thích các bước giải:

O là trung điểm AB=>OA=OB=3(cm)

Th1 M€OA:

+OM=1(cm)

->+AM=OA-OM=3-1=2(cm)

   +MB=OM+OB=1+3=4(cm)

Th2: M€ OB:

+OA=4(cm)

+OB=2(cm)

a: Các đoạn thẳng là AM,MB,AB

b: M là trung điểm của AB

=>\(MB=MA=\dfrac{AB}{2}=\dfrac{6}{2}=3\left(cm\right)\)

c: Ta có: K nằm trên đoạn MA

=>K nằm giữa A và M

=>AK+KM=AM

=>KM=AM-AK=3-1=2cm

23 tháng 11 2017

viết tập hợp A={XEN*/X <5}

18 tháng 8 2017

              1

 l----------l-----l---------------l

A        M    O             B

ta có : AB=6 => AO,BO = 1/2 AB = 3 

Mà OM = 1 => AM = 3 - 1 = 2 cm

                      BM = 3 + 1 = 4 cm

27 tháng 7 2018

Hai trường hợp nha bạn

a: C nằm giữa A và B

=>CA+CB=AB

=>CB=4cm

b: C nằm giữa A và B

CA=CB

=>C là trung điểm của AB

21 tháng 2 2018

a) 

Vì M thuộc AB nên AM + MB = AB

AM + 2 = 5  AM = 3 cm

Có AN = AM  AN = 3 cm

Do N thuộc tia đối của tia AB nên điểm A nằm giữa N và B

BN = AB + AN = 5 + 3 =  8 cm

b)

+ Trên cùng một nửa mặt phẳng có bờ chứa tia AB có:    Tia Ax nằm giữa hai tia AB và Ay nên ta có:  

hay

+   Trên cùng một nửa mặt phẳng có bờ AB, ta có  và  là hai góc kề bù .

 

hay     

c)

Vì BN = AB + AN = 5 + AN

 Suy ra BN có độ dài lớn nhất khi AN có độ dài lớn nhất

Mà AN = AM  BN có độ dài lớn nhất khi AM có độ dài lớn nhất

 Có AM  AB  AM lớn nhất khi AM = AB khi đó điểm M trùng với điểm B.

Vậy khi điểm M trùng với điểm B thì BN có độ dài lớn nhất.

30 tháng 12 2018

Vẽ hình như nào hả bạn